LSAT
PrepTest 94
Law School Admission Council
1 -2-
1 1
SECTION I
Time—35 minutes
23 Questions
Directions: Each set of questions in this section is based on a scenario with a set of conditions. The questions are to be
answered on the basis of what can be logically inferred from the scenario and conditions. For each question, choose the
response that most accurately and completely answers the question and mark that response on your answer sheet.
Questions 1–5 1. Which one of the following could be an accurate
assignment of security badges to employees?
A company has installed a new security system and is
assigning security badges to the following employees: Flores, (A) level 1 badge: none
Gonzales, Henderson, Wong, Yamamoto, and Zamora. Of level 2 badge: Flores, Henderson
these, Flores, Gonzales, and Henderson are recent hires; the level 3 badge: Gonzales, Yamamoto
others are experienced. Wong and Zamora are managers; the level 4 badge: Wong, Zamora
others are not. There are four levels of security badges, from (B) level 1 badge: Henderson
level 1 (the lowest) through level 4 (the highest). Badges will level 2 badge: Flores, Gonzales, Yamamoto
be assigned to employees in accordance with the following level 3 badge: Wong
rules: level 4 badge: Zamora
Every manager must be assigned a badge of a (C) level 1 badge: Flores, Gonzales
higher security level than any badge assigned to a level 2 badge: Henderson
nonmanagerial employee. level 3 badge: Yamamoto, Zamora
Every experienced employee must be assigned a badge level 4 badge: Wong
of a higher security level than any badge assigned to a (D) level 1 badge: Gonzales, Henderson
recent hire. level 2 badge: Flores
level 3 badge: Yamamoto
level 4 badge: Wong, Zamora
(E) level 1 badge: Flores, Gonzales, Henderson
level 2 badge: Wong
level 3 badge: Yamamoto
level 4 badge: Zamora
GO ON TO THE NEXT PAGE.
Law School Admission Council
1 1 -3-
1
2. If none of the employees is assigned a level 1 badge, 4. If none of the employees is assigned a level 4 badge,
which one of the following employees must be assigned a which one of the following must be true?
level 3 badge?
(A) Henderson is assigned a level 2 badge.
(A) Gonzales (B) Yamamoto is assigned a level 3 badge.
(B) Henderson (C) Flores is assigned a different level badge than
(C) Wong Gonzales.
(D) Yamamoto (D) Flores is assigned a different level badge than
(E) Zamora Henderson.
(E) Wong is assigned the same level badge as
3. If exactly two of the recent hires are assigned level 2 Zamora.
badges, which one of the following could be true?
5. Which one of the following, if substituted for the rule that
(A) Flores is assigned a higher level badge than
every experienced employee must be assigned a badge
Gonzales.
of a higher security level than any badge assigned to a
(B) Wong is assigned a higher level badge than
recent hire, would have the same effect in determining
Zamora.
the assignment of badges to employees?
(C) None of the employees is assigned a level 1
badge. (A) Yamamoto must be assigned a badge of a higher
(D) None of the employees is assigned a level 3 security level than any badge assigned to a
badge. recent hire.
(E) None of the employees is assigned a level 4 (B) Every manager must be assigned a badge of a
badge. higher security level than any badge assigned to
a recent hire.
(C) Each experienced employee must be assigned
either a level 3 badge or a level 4 badge.
(D) If none of the employees is assigned a level 1
badge, then Flores, Gonzales, and Henderson
must all be assigned level 2 badges.
(E) If none of the employees is assigned a level 4
badge, then Flores, Gonzales, and Henderson
must all be assigned level 1 badges.
GO ON TO THE NEXT PAGE.
Law School Admission Council
1 -4-
1 1
Questions 6–11 6. Which one of the following could be a complete and
accurate list of the courses the student takes in each
A student takes exactly six one-semester courses—geology, semester?
history, literature, mathematics, physics, and religion. Each
course is taken in exactly one of two semesters—semester 1 (A) semester 1: geology, history, literature
or semester 2. The following conditions apply: semester 2: mathematics, physics, religion
The student takes mathematics and physics in the same (B) semester 1: geology, literature, religion
semester. semester 2: history, mathematics, physics
It is not the case that the student takes both physics and (C) semester 1: literature, mathematics, physics
religion in semester 2. semester 2: geology, history, religion
If the student takes history in semester 2, she takes (D) semester 1: geology, literature, mathematics,
geology and literature in semester 1. physics
If the student takes mathematics and geology in semester semester 2: history, religion
1, she takes literature in semester 2. (E) semester 1: history, literature, mathematics,
religion
semester 2: geology, physics
GO ON TO THE NEXT PAGE.
Law School Admission Council
1 1 -5-
1
7. Which one of the following could be a complete 10. If the student takes physics and religion in the same
and accurate list of the courses the student takes in semester as each other, then which one of the following
semester 1? courses could the student take in semester 2?
(A) history (A) geology
(B) geology, religion (B) history
(C) literature, mathematics (C) mathematics
(D) geology, mathematics, religion (D) physics
(E) geology, history, literature, religion (E) religion
8. If the student takes geology in semester 2, then each of 11. If the student takes literature in semester 1, then which
the following could be true EXCEPT: one of the following must be true?
(A) The student takes exactly two courses in (A) The student takes history in semester 2.
semester 1. (B) The student takes mathematics in semester 2.
(B) The student takes exactly four courses in (C) The student takes religion in semester 1.
semester 1. (D) The student takes at least three courses in
(C) The student takes history in semester 2. semester 1.
(D) The student takes mathematics in semester 2. (E) The student takes at most three courses in
(E) The student takes religion in semester 1. semester 1.
9. Which one of the following could be the only course that
the student takes in semester 2?
(A) history
(B) literature
(C) mathematics GO ON TO THE NEXT PAGE.
(D) physics
(E) religion
Law School Admission Council
1 -6-
1 1
Questions 12–16 12. Which one of the following could be the order, from first
to last, in which the floats appear?
The organizers of a small-town holiday parade must
determine the order in which six floats will appear as they (A) fire department, hardware store, library, grocery
pass consecutively in front of the town hall. Each float is store, youth center, kennel
sponsored by a different business or organization—the fire (B) fire department, library, grocery store, hardware
department, the grocery store, the hardware store, the kennel, store, kennel, youth center
the library, and the youth center. The following constraints (C) kennel, library, grocery store, hardware store, fire
apply: department, youth center
The library float must appear immediately before the (D) library, fire department, grocery store, hardware
grocery store float. store, youth center, kennel
The grocery store float must appear earlier than the (E) library, grocery store, hardware store, youth
hardware store float, and the hardware store float must center, kennel, fire department
appear earlier than the youth center float.
The hardware store float cannot appear immediately
before or immediately after the kennel float.
The fire department float cannot appear immediately
before or immediately after the youth center float.
GO ON TO THE NEXT PAGE.
Law School Admission Council
1 1 -7-
1
13. If the grocery store float appears immediately before 15. The order in which the floats appear is fully determined if
the kennel float, then which one of the following must which one of the following is true?
be true?
(A) The fire department float appears third.
(A) The fire department float appears first. (B) The grocery store float appears fourth.
(B) The library float appears second. (C) The hardware store float appears fourth.
(C) The kennel float appears third. (D) The kennel float appears fifth.
(D) The hardware store float appears fourth. (E) The library float appears third.
(E) The youth center float appears fifth.
16. The kennel float must appear last if which one of the
14. The fourth float to appear CANNOT be that of following is true?
(A) the fire department (A) The fire department float appears first.
(B) the grocery store (B) The library float appears second.
(C) the hardware store (C) The grocery store float appears third.
(D) the kennel (D) The hardware store float appears fourth.
(E) the youth center (E) The youth center float appears fourth.
GO ON TO THE NEXT PAGE.
Law School Admission Council
1 -8-
1 1
Questions 17–23 17. Which one of the following could be a complete and
accurate list of the books purchased by the collector?
A book collector is trying to decide which of the following six
books to purchase at an auction—a literary classic, a mystery, (A) literary classic, play
a play, a romance novel, a science fiction book, and a tragedy. (B) literary classic, tragedy
The book collector’s purchase must be made according to the (C) mystery, play, science fiction book
following considerations: (D) mystery, play, romance novel, tragedy
If both the literary classic and the mystery are purchased, (E) literary classic, mystery, romance novel, science
then the play is purchased. fiction book
If either the play or the science fiction book is purchased,
then the romance novel is not purchased.
If the tragedy is not purchased, then both the mystery and
the romance novel are purchased.
GO ON TO THE NEXT PAGE.
Law School Admission Council
1 1 -9-
1
18. If the collector purchases exactly two books, which one 21. If the book collector purchases the mystery, then which
of the following must be true? one of the following must be true?
(A) The mystery is purchased. (A) The play is purchased.
(B) The tragedy is purchased. (B) The romance novel is purchased.
(C) The romance novel is not purchased. (C) The tragedy is purchased.
(D) The literary classic and the mystery are not both (D) At least two books are purchased.
purchased. (E) At most two books are purchased.
(E) The romance novel and the tragedy are not both
purchased. 22. If the book collector purchases the play, then which one
of the following must be true?
19. If the book collector purchases neither the play nor the
(A) The tragedy is purchased.
science fiction book, then each of the following could be
(B) The mystery is purchased.
true EXCEPT:
(C) The literary classic is purchased.
(A) The romance novel is purchased. (D) Exactly two books are purchased.
(B) The tragedy is not purchased. (E) Exactly three books are purchased.
(C) Exactly one book is purchased.
(D) Exactly three books are purchased. 23. If the book collector purchases the romance novel, then
(E) Exactly four books are purchased. which one of the following could be true?
(A) Both the literary classic and the mystery are
20. Which one of the following, if true, completely
purchased.
determines which books are purchased by the collector?
(B) Exactly three books are purchased.
(A) The romance novel is not purchased. (C) Exactly four books are purchased.
(B) The tragedy is not purchased. (D) The play is purchased.
(C) Neither the literary classic nor the mystery is (E) The science fiction book is purchased.
purchased.
(D) Neither the mystery nor the romance novel is
purchased.
(E) Neither the play nor the science fiction book is
purchased.
S T O P
IF YOU FINISH BEFORE TIME IS CALLED, YOU MAY CHECK YOUR WORK ON THIS SECTION ONLY.
DO NOT WORK ON ANY OTHER SECTION IN THE TEST.
Law School Admission Council
2 -10-
2
SECTION II
2 2
Time—35 minutes
25 Questions
Directions: Each question in this section is based on the reasoning presented in a brief passage. In answering the questions, you
should not make assumptions that are by commonsense standards implausible, superfluous, or incompatible with the passage. For
some questions, more than one of the choices could conceivably answer the question. However, you are to choose the best
answer; that is, choose the response that most accurately and completely answers the question and mark that response on your
answer sheet.
1. When primatologist Akira Suzuki began studying snow 3. A recent study has found that, surprisingly, the risk of
monkeys in the 1950s, he found that they often roamed serious injuries to workers is higher in industries that
out of the mountains to feed in apple orchards. After a are monitored by government safety inspectors than in
decade of observing this behavior, Suzuki began to feed industries that are not so monitored.
the monkeys in their mountain habitat by providing them Which one of the following, if true, most helps to explain
with soybeans to eat. The monkeys no longer raided the the surprising finding described above?
orchards. When Suzuki began his work, 23 snow monkeys
lived in the region he studied. The population today is (A) Government safety inspectors not only monitor
270 snow monkeys and is expected to continue growing. but also train employees of the inspected firms to
follow safe practices.
Which one of the following claims is most strongly (B) Government safety inspectors do not have the
supported by the information above? authority to enforce safety regulations.
(A) Snow monkeys do not feed outside of their (C) Only those industries with an inherently high risk
mountain habitat when food is readily available of on-the-job injury are monitored by government
within it. safety inspectors.
(B) For snow monkeys, soybeans provide more (D) Workers behave especially cautiously when they
complete nutrition than other beans. believe their performance is being monitored by
(C) In feeding soybeans to the monkeys, Suzuki did government safety inspectors.
not intend to provoke the phenomenal population (E) Some of the industries that are monitored by
growth that resulted. government safety inspectors have much lower
(D) Snow monkeys eat apples only if there is no other rates of injuries than do other industries that are
fruit to eat. also so monitored.
(E) Feeding soybeans to snow monkeys has proved to
be an environmentally unsound policy. 4. Business writer: Although the demand for Corinne
wristwatches currently far outstrips supply, it
2. Body temperature can be estimated by the relative would be a mistake for the manufacturer to increase
proportion of two oxygen isotopes in bone. Such estimates supply to meet demand. The current demand results
derived from dinosaur fossils suggest that dinosaurs had from the public’s perception that the wristwatches
nearly the same body temperature in their limbs as in the are in short supply, and the wristwatches are in
rest of their bodies. Typically, the core body temperature of short supply merely because they are produced
cold-blooded animals today is much higher than the body in very limited quantities. The excess demand
temperature in their limbs. Thus, dinosaurs were probably creates the impression that the wristwatches are
warm-blooded. greatly desired, and that impression in turn helps
account for consumers’ desire for them. If the
Which one of the following, if true, most seriously supply of Corinne wristwatches were to increase
weakens the argument? to meet demand, excess demand for them would
(A) Large warm-blooded animals keep their core be eliminated, with the result that the wristwatches
body temperature slightly higher than the body would no longer be desired.
temperature in their limbs. The claim that it would be a mistake for the manufacturer
(B) The fossilization process introduces changes to of Corinne wristwatches to increase supply to meet
bones such that their original oxygen isotope demand plays which one of the following roles in the
ratios cannot be predicted. business writer’s argument?
(C) Oxygen was more abundant in Earth’s atmosphere
during the period in which the dinosaurs lived. (A) an introductory claim that describes the position to
(D) Small warm-blooded animals like mice tend to be refuted by the argument
have more uniform body temperatures than do (B) a justification of the relevance of the evidence cited
large warm-blooded animals like elephants. (C) a premise of the argument
(E) Warm-blooded animals are more active and use (D) an opinion offered in support of one of the
more oxygen than cold-blooded animals. argument’s premises
(E) the conclusion toward which the argument is
directed
GO ON TO THE NEXT PAGE.
Law School Admission Council
2 5.
2
Zobel: Peterson’s analytic concepts are wrong and should 6.
2
Some psychologists claim that empathic responses are
-11-
2
be rejected. As a psychoanalyst myself, however, forms of moral behavior. Having observed that young
I can understand why certain psychoanalysts children who witness another’s distress respond by
adhere to them. These psychoanalysts acquired expressing sadness and offering help, these psychologists
their “emotional certainty” that Peterson’s views believe that moral behavior begins early in life. A second
are correct while training under her. This training group of psychologists claims that empathic response is
includes one’s own psychoanalysis, in which not, by itself, moral behavior and that in order to count as
the teacher interprets the actions, dreams, and moral, behavior must be based on a clear understanding
fantasies of the student in analytic terms. Strong of moral principles and a certain degree of moral
emotional bonds with the teacher are formed, reasoning skill. On the basis of children’s unsophisticated
bonds that predispose the student to accept verbal responses to hypothetical moral dilemmas, these
the teacher’s analytic concepts as correct. It is psychologists conclude that children lack the degree
thus impossible for a student to make unbiased of moral reasoning skill necessary for their behavior,
judgments about the value of the teacher’s however compassionate, to be considered moral.
analytic concepts.
Which one of the following, if true, most seriously
Based on the information in the passage, which one of undermines the conclusion drawn by the second group
the following is the most accurate assessment of Zobel’s of psychologists?
claim that Peterson’s analytic concepts are wrong and
(A) The children studied by the second group of
should be rejected?
psychologists displayed a slightly higher level
(A) The claim is dubious because Zobel assumes of moral reasoning when they were well rested
without justification that she is qualified to judge than when they were tired.
Peterson’s analytic concepts. (B) Adults who respond to hypothetical moral
(B) The claim has not been established because Zobel dilemmas display a much higher level of moral
provides evidence that could show only that reasoning than do children who responded to the
Peterson’s students are biased in the evaluation same hypothetical moral dilemmas.
of Peterson’s analytic concepts. (C) The children studied by the second group of
(C) The claim cannot be evaluated because it is not psychologists displayed a slightly higher level
possible for any psychoanalyst to provide an of moral reasoning in response to hypothetical
objective assessment of another psychoanalyst’s dilemmas involving adults than in response to
analytic concepts. hypothetical dilemmas involving children.
(D) The claim is questionable because it is obvious (D) In actual situations involving moral dilemmas,
that Zobel has a professional rivalry with children display a much higher level of moral
Peterson and cannot judge Peterson’s concepts reasoning than did the children who, in the study
fairly. by the second group of psychologists, responded
(E) The claim is acceptable because Zobel has only to hypothetical dilemmas.
effectively shown that Peterson’s analytic (E) Some adults who respond to hypothetical moral
concepts are biased and based on emotion rather dilemmas reason at about the same level as
than rational thinking. children who respond to the same hypothetical
moral dilemmas.
GO ON TO THE NEXT PAGE.
Law School Admission Council
2 -12-
7.
2
Mr. Xu: The arctic squirrel gets so cold while hibernating 9.
2
Editorialist: Evidence shows that restrictions on tobacco
2
that its blood temperature falls well below the advertising have had a significant impact on
temperature at which water freezes. Although smoking among adults. A recent survey has
the squirrel’s blood is about 70 percent water, shown that a smaller percentage of adults now
the blood never freezes while the squirrel is smoke than at any other time in the last two
hibernating. The squirrel’s blood, therefore, decades. The decline in the percentage of adults
must contain a substance that prevents the blood who smoke has been most marked during the last
from freezing at the temperature at which water ten years, and, not coincidently, some of the most
freezes. important restrictions on tobacco advertising
Ms. Yerky: The blood contains no such substance. came into force ten years ago.
Laboratory experiments involving a number of The reasoning in the editorialist’s argument is most
animals, including the arctic squirrel, have shown vulnerable to criticism on the grounds that the argument
that a vial of blood from any of the animals
freezes at just the same temperature as does a vial (A) fails to consider whether there have been any
of water. changes over the last two decades in the
percentage of the teenage population who smoke
Ms. Yerky’s response proceeds by (B) uses evidence that describes only a percentage of
(A) presenting evidence that supports a conclusion the adult population to reach a conclusion about
inconsistent with Mr. Xu’s conclusion the entire adult population
(B) showing that the evidence offered by Mr. Xu was (C) reaches a conclusion about smoking among
collected by means of unreliable methods today’s adults based on statistics from ten or
(C) offering an alternative explanation for why the twenty years ago
squirrel’s blood fails to freeze at the temperature (D) neglects to take into account whether there have
at which water freezes been restrictions on the advertising of other
(D) showing that a key term used by Mr. Xu is products besides tobacco in the past ten years
ambiguous (E) fails to consider the possibility that factors other
(E) showing that the evidence provided by Mr. Xu than restrictions on advertising have contributed
has no bearing on the point at issue to the decline in smoking among adults
8. Historian: A democracy’s citizens must know some
history if the democracy is to meet its challenges.
However, popular historical awareness is
inevitably distorted, for most people learn history
through popular narratives that sustain readers’
GO ON TO THE NEXT PAGE.
interest by implying that a few famous heroes and
notorious villains have shaped all of history.
The historian’s argument depends on assuming which
one of the following?
(A) Historical awareness is distorted by the view that
there have been only a few famous heroes or
notorious villains.
(B) History cast in the narrative format inevitably
distorts historical reality.
(C) Most historical narratives sustain interest
by implying that a few famous heroes and
notorious villains have shaped all of history.
(D) Only narratives written for a purpose other
than sustaining readers’ interest can convey an
undistorted awareness of history.
(E) The implication that a few famous heroes and
notorious villains have shaped all of history
distorts history.
Law School Admission Council
2 2
10. Medical researcher: Studies in North America have
2 -13-
12. Politician: Tightening air quality standards that regulate
2
shown that the incidence of heart disease in a industrial emissions would cause industries to
population is closely related to the average fat move to locations with less stringent standards
consumption for individuals in that population. concerning these emissions. So current standards,
However, although residents of France which are already quite stringent, should not be
consume, on average, as much fat as residents raised, since there is not enough evidence that the
of North America, heart disease presently occurs decreased pollution that would admittedly result
half as frequently among the French as among from raising the standards would compensate for
North Americans. the loss of jobs caused by the relocation of key
industries.
Which one of the following, if true, would most help to
resolve the apparent conflict noted by the medical Which one of the following principles, if valid, most
researcher? helps to justify the politician’s reasoning?
(A) The average level of fat consumption by the (A) Governmental policy should generally be
French has been falling for several decades. designed to encourage the growth of key
(B) Other factors of diet besides high consumption of industries.
fat have not been similarly linked with incidence (B) The extent to which the people are willing to
of heart disease. accept a governmental policy should be the
(C) Heart disease takes years to develop and the only factor determining whether that policy is
average level of fat consumption in France adopted.
increased to North American levels only a few (C) Governmental policy should be altered only
years ago. if there is compelling evidence that the
(D) Certain diseases other than heart disease have consequences of doing so are better than the
also been linked to average fat consumption, and consequences of not doing so.
the French have a higher incidence of these than (D) Governmental policy should be changed in those
do North Americans. cases in which retaining the policy and changing
(E) Cigarette smoking significantly increases the it each have positive and negative consequences.
risk of heart disease and France has a higher (E) If one lacks clear evidence about whether a
percentage of cigarette smokers in its population given action will have a specified consequence,
than does North America. then one should assume that it will have that
consequence when deciding whether to perform
11. Professor Williams: Professor Thomas’s report that action.
characterizes our colleague Professor York
as too flamboyant and confrontational in the 13. Santayana recommends that we study history to avoid the
classroom. But the argument given in this report mistakes of the past. But we should not follow his advice.
sounds so much like the classic argument of the For, since history consists of unique and unrepeatable
self-serving academic that one has to wonder accidents, none of the crises we now face are the same as
if Professor Thomas simply is not the orator those our ancestors faced. Thus, studying history never
and entertainer Thomas would like to be, and enables one to avoid mistakes of the past.
so vents frustrations by condemning others—
Which one of the following most accurately expresses
Professor York in particular.
the main conclusion of the argument above?
Professor Williams’s argument is flawed because it
(A) People should not study history, since doing so
(A) confuses the distinction between being overly leads them to misunderstand the crises they
confrontational and engaging students by now face.
entertaining them (B) Every historical period is different from every
(B) presupposes the point it is attempting to establish other historical period.
(C) mistakes Professor Thomas’s characterization of (C) Although the crises one generation faces
a view for an endorsement of that view may appear to be the same as those another
(D) attacks Professor Thomas personally rather than generation faces, they never are.
addressing Professor Thomas’s argument (D) Studying history is valuable, but not for the
(E) rejects the possibility that Professor York is in reason that Santayana suggests.
fact too confrontational (E) One should not try to avoid repeating the
mistakes of previous generations by studying
history.
GO ON TO THE NEXT PAGE.
Law School Admission Council
2 -14-
2
14. When polar ice caps grow (during ice ages, for example),
2
16. A recent study found that small rats were approximately
2
lighter forms of oxygen from water vapor and seawater twice as likely, and large rats only half as likely, to suffer
accumulate in the frozen ice caps, leaving greater from heart problems than were rats of average size.
concentrations of one heavy form of oxygen behind in
Each of the following, if true, contributes to an
the sea, where it is absorbed by marine organisms. When
explanation of the correlation given above between size
and as the ice caps shrink, the concentrations of this
and heart disease in rats EXCEPT:
heavy oxygen in seawater decrease. During one 30,000-
year period, concentrations of this heavy oxygen in sea (A) Small rats are more likely than large rats to suffer
shellfish increased for about 20,000 years, then decreased from fatal diseases that tend to strike earlier than
for 10,000 years. do heart problems.
(B) Small rats generally have smaller blood vessels
The information given most strongly supports which one
than do large rats, vessels that can more easily
of the following hypotheses about the period described
be clogged with fatty deposits.
above?
(C) The larger a rat is, the more successful it will
(A) Average global temperatures 10,000 years after be at defending itself, and therefore the less
the beginning of the period approximately stressful its existence will be.
equaled average global temperatures (D) In addition to being the leading cause of weak
20,000 years later. hearts in adult rats, malnutrition at early stages
(B) Polar ice caps at the beginning of the period were of development causes rats to be undersized.
larger than they were at the end of the period. (E) Although large rats are no more active than rats
(C) The beginning of the period coincided with the of average size, they tend to be much more
onset of an ice age that lasted approximately active than small rats, resulting in greater
20,000 years. cardiovascular conditioning.
(D) The polar ice caps grew for about 20,000 years
after the period began, then began to shrink. 17. To measure a small boat hull accurately, it is necessary to
(E) An ice age was drawing to an end during the first level the hull so that a baseline can be established. This
20,000 years of the period. does not require sophisticated tools, but it does require
a leveling tool called a “line level.” When scientists
15. People would not follow a leader if they felt that there discovered a small ninth-century boat, they wanted to
was nothing they could gain by following that leader. record the dimensions of its hull accurately. None of the
Therefore, even those leaders who are incompetent or scientists, however, had any experience measuring hulls.
evil bring some good to their followers. Because of this they were not sure they had leveled the
hull before establishing a baseline.
Which one of the following is most similar in its flawed
reasoning to the flawed reasoning in the argument If the statements above are true, then which one of the
above? following must also be true?
(A) Because people expound upon only theories they (A) The scientists did not accurately measure
believe are true, any theory that is expounded the hull’s dimensions because they had no
upon by someone contains at least a grain of experience measuring hulls.
truth. (B) The scientists accurately determined the hull’s
(B) Because there is some good to be found in even dimensions, provided that they leveled the hull.
the worst circumstances, the world’s most (C) If the scientists did not accurately determine the
vicious people must have brought about at least hull’s dimensions, it was because they did not
some good. have a line level.
(C) To be a worthy leader, one must bring some good (D) The scientists were able to accurately record the
to people. Therefore, those who bring about hull’s dimensions only if they used a line level.
some good make worthy leaders. (E) If the scientists had measured the hull’s
(D) Because people can never completely separate dimensions accurately, then at least one of them
what is true from what they wish to be true, no would have had experience measuring hulls.
theory is ever completely true.
(E) Even leaders of scientific revolutions are
influenced by the false theories of their
predecessors. Therefore, the influence of earlier
false theories never completely disappears.
GO ON TO THE NEXT PAGE.
Law School Admission Council
2 2
18. Dapolito: The city council recently passed a rent-control
2
20. Economist: When national governments dispense funds
-15-
2
ordinance. But a recent study of fifteen local to local governments to spend on local projects,
communities shows clearly that rent control any local government can receive a greater
increases the price and lowers the quality proportion of government funds by creating more
and availability of rental units. Thus, it can local projects than other local governments create
be concluded that it is not the city council’s on average. Due to this added incentive to create
objective to preserve the quality and availability more local projects, overall government spending
of local rentals. and taxation are greater everywhere than they
would be if local governments funded projects
Dapolito’s conclusion follows logically if which one of
entirely by themselves.
the following is assumed?
Which one of the following most closely conforms to the
(A) The recent study of local rent-control ordinances
proposition illustrated by the economist’s statements?
was conducted by impartial investigators.
(B) Rent control is not an appropriate topic for (A) A large company invests in a new technology
consideration by the city council. that greatly improves its product. Smaller
(C) The members of the city council who voted for companies, individually unable to match
rent control agree with the study’s conclusions the large company’s investment, pool their
about rent control. resources and invest in the new technology in
(D) Some members of the city council who voted for order to compete.
rent control stand to profit from rent control. (B) A national government finances an irrigation
(E) The city council sometimes acts in an arbitrary project in order to turn an arid valley into fertile
and irrational manner. farmland. The food grown in the valley reduces
local prices, which helps consumers in the
19. Geologist: As a spinning object’s radius decreases, its rate valley but hurts farmers elsewhere.
of rotation increases; for example, figure skaters (C) A sales manager offers prizes for the salesperson
can twirl faster by bringing their arms closer to who sells the most products during a month.
their sides. This law of physics helps to explain This causes each salesperson to try to undercut
why, over the past decade, Earth’s rate of rotation the prices of the other salespeople, and as
increased. For earthquakes have increased in a result profits are lower than they would
number and severity during the past decade; and otherwise be.
since immediately after an earthquake Earth’s (D) People pool their money to buy certain foods
tectonic plates move toward the center of the in bulk in order to get a volume discount and
earth, their movement causes Earth’s radius to reduce their bills. But after doing this, they eat
decrease. more than they used to and so spend the same
Which one of the following, if true, would most amount of money on food as they did before
undermine the geologist’s explanation of why Earth’s they pooled it.
rate of rotation increased over the last decade? (E) Several communities band together to build a
large conference center. But the project nearly
(A) In the beginning of the twentieth century, collapses as each community, because of the
geologists observed an increase in number and revenue the center will bring, fights to be the
severity of earthquakes, accompanied by a one in which it is built.
decrease of Earth’s radius.
(B) During the past decade, other geological events
have counteracted the movements of the tectonic
plates that occur immediately after earthquakes.
(C) Only skillful figure skaters succeed in twirling
faster by bringing their arms closer to their
sides. GO ON TO THE NEXT PAGE.
(D) Since the time of the ancient Egyptians, Earth’s
rate of rotation has been known to fluctuate.
(E) Increased volcanic activity over the last ten
years suggests that the overall movement of the
tectonic plates has increased.
Law School Admission Council
2 -16-
2
21. Philosopher: For some kinds of art, there is truth to the
2
23. When a resource’s price reflects its full cost to society,
2
adage that beauty is in the eye of the beholder. consumers pay directly for externalities—the unintended
A painting or sculpture that strikes one person but harmful consequences to society of using a resource.
as having artistic value may strike another as Further, the price of a resource deters misuse of that
lacking such value. Consequently, there are no resource only if it reflects the resource’s externalities.
valid objective standards for determining the So, since sound management of a resource requires that
artistic value of a painting or a sculpture. the price of the resource deter its misuse, it also requires
that the price reflect whatever unintended harm to the
The philosopher’s reasoning is flawed because it
environment the use of that resource causes.
(A) relies exclusively on an old adage whose truth
The conclusion follows logically if which one of the
has not been demonstrated
following is assumed?
(B) ignores the fact that there are other kinds of art,
such as literature and poetry, that may also (A) Whatever constitutes unintended harm to the
occasion disagreement environment also constitutes unintended harm to
(C) fails to consider whether there are valid objective society.
standards for evaluating nonvisual art, such as (B) A resource’s externalities are always taken into
music account in setting the price of that resource.
(D) fails to consider that people who disagree about (C) When setting a resource’s price, it is possible to
the artistic value of a given painting may be forecast completely and accurately all of the
incorrectly applying the same evaluation criteria effects of using that resource.
to that painting (D) If a resource is soundly managed, then its
(E) fails to consider that the individuals judging and externalities are precisely assessed.
disagreeing about works of art may be experts in (E) The price of a given resource is only one of
making such judgments several factors relevant to a decision about
whether to use it.
22. Researcher: Experiences that are accompanied by
increased secretions of adrenaline—a hormone
produced in situations involving fear—tend to be
remembered more clearly than experiences not
so accompanied. Thus, the details of frightening
experiences tend to be remembered more clearly
than do the details of nonfrightening experiences. GO ON TO THE NEXT PAGE.
Which one of the following, if true, most seriously
weakens the researcher’s argument?
(A) Some experiences are so intense that an
individual’s normal tendency to retain the details
of them is reversed.
(B) An individual will tend to remember most clearly
those details of a situation that are relevant to
the satisfaction of desires.
(C) Highly pleasurable experiences are, like
frightening experiences, accompanied by
increased levels of adrenaline.
(D) Frightening experiences make up only a small
fraction of experiences in general.
(E) If an individual perceives a dangerous situation
as nonfrightening, then the experience of that
situation will not be accompanied by increased
adrenaline secretions.
Law School Admission Council
2 2
24. Researcher: Several chemical compounds found
2
25. According to the Newtonian theory of gravity, the
-17-
2
in soybeans are effective agents for cancer gravitational force between two bodies depends
prevention in humans. These include isoflavones exclusively on their mass and the distance between them,
and phytosterols, which also occur in other plants but is independent of what the bodies are made of. Thus,
but are especially concentrated in soybeans. In according to the Newtonian theory, one does not need a
rats and larger mammals such as cheetahs and theory of the structure and constitution of the Sun and the
sheep, isoflavones have been shown to inhibit the planets in order to calculate their orbits.
production of estrogen, and estrogen is known
The conclusion of the argument above follows logically
to promote certain types of cancer. Phytosterols
if which one of the following is assumed?
may slow cholesterol absorption and thus have
anticancer effects. (A) Mass is dependent on gravitational force.
(B) According to the Newtonian theory, the
Each of the following, if true, would reduce the support
calculation of planetary orbits requires
given by the researcher for the claim that several
considering only their gravitational forces.
chemical compounds in soybeans are effective agents for
(C) The Newtonian theory of gravity is mistaken
human cancer prevention EXCEPT:
about the data needed to calculate gravitational
(A) The soybean derivatives used in most studies force.
are the products of specialized processing (D) Knowing what an object is made of is sufficient
techniques and are not yet widely available to for determining its mass.
consumers. (E) The gravitational force between the Sun and a
(B) While phytosterols, which occur in high planet is a factor in determining the distance
concentrations in soybeans, have been shown to between them.
decrease cholesterol absorption in the body, new
evidence suggests that this decrease is not large
enough to reduce susceptibility to cancer.
(C) A study of people with high levels of blood
cholesterol showed no significant reduction in
cholesterol levels after switching to a soybean-
rich diet.
(D) Consumption of soybean products might lower
blood cholesterol in animals, but a study
of people whose major source of protein is
soybeans rather than animal products showed
blood cholesterol levels no lower than normal.
(E) Preliminary studies have not shown isoflavones
to have a significant effect on estrogen levels in
humans.
S T O P
IF YOU FINISH BEFORE TIME IS CALLED, YOU MAY CHECK YOUR WORK ON THIS SECTION ONLY.
DO NOT WORK ON ANY OTHER SECTION IN THE TEST.
Law School Admission Council
3 -18-
3 3 3 3
SECTION III
Time—35 minutes
27 Questions
Directions: Each set of questions in this section is based on a single passage or a pair of passages. The questions are to be
answered on the basis of what is stated or implied in the passage or pair of passages. For some questions, more than one of the
choices could conceivably answer the question. However, you are to choose the best answer; that is, choose the response that
most accurately and completely answers the question and mark that response on your answer sheet.
Two competing demands we make of the law law can adapt to social reality. The attempt to reconcile
create a troubling conflict that contributes to the law’s formalism and substantive justice within a legal system
frequent failure to deliver what we imagine it should. situated in conditions of social inequality is a misguided
On one hand, we want a formalist system of law—one project.
that is rational and rule-based, and that promises to
deliver us from arbitrariness, irrationality, and caprice 1. Which one of the following would, if true, most seriously
in a coherent, reasoned way. It should be a system undermine the author’s conclusion about formalism in
that we are capable of understanding and generally legal systems?
able to depend on so that we can order our affairs with (A) The vast majority of people are quite aware of the
the knowledge that none of us will be individually imperfections of merely procedural justice but
disadvantaged before it. In short, it is imperative that do not see any practical way of remedying these
the law provide a universal ordering principle that is imperfections.
rational, consistently applied, and blind to the differing (B) Nonformalist legal systems have been found to
social and economic situations of those who come deliver substantive justice even less frequently
before it. than their formalist counterparts.
On the other hand, however, we want the law to be (C) Any type of collective action that is formalized
connected to social reality and sensitive to the particular into a set of rules tends to become less effective
contexts out of which individual cases arise. After all, as those rules become dated and less relevant.
a body of law is of little use if it delivers justice that is (D) Societies in which there is little social inequality
merely procedural as opposed to substantive—i.e., if tend to use formalism as a basis for their legal
it cannot deliver results in individual cases that we can systems.
recognize as fair and equitable. It need not, of course, (E) A formalist approach to the law is sometimes
be the outcome that we, as individual parties, would found to be too broad and abstract to apply to
want; someone must, after all, lose in an adversarial the particulars of individual cases.
system. But it must at least be an outcome that we,
as parties and observers, can accept as legitimate and 2. Which one of the following most accurately expresses the
justifiable according to some common understanding of message the author intends to convey in using the phrase
fairness and justice. Cases should not, for example, be “misguided project” in the final sentence of the passage?
decided based strictly on the application of unalterable
rules without regard for the equity or reasonableness of (A) The project would be much more difficult than
the outcome. Such extreme formalism, applied neutrally the benefits of its success would warrant.
and without prejudice but also without sensitivity to the (B) Such a project is bound to produce unforeseen
social contexts of those involved, speaks little to the negative consequences.
substance of justice. And it is imperative that the legal (C) The project lacks the expert guidance that would
system deliver substantive justice in individual cases— be necessary to make it a success.
that it derive from and relate to subjective experience as (D) The steps necessary to make such a project
well as objective reason. succeed would violate several important
It seems, then, that we are left with an principles of justice.
irreconcilable tension arising from the competing (E) Such a project cannot, given the present context,
imperatives of formalism and substantive justice. be successful.
Any attempt to accommodate both principles within
the legal system is bound to fail for systemic reasons:
formalism cannot produce substantive justice until
there is a reasonable degree of social equality. Without
this equality, formalism may very well perpetuate
injustice; no universal rules can guarantee otherwise. GO ON TO THE NEXT PAGE.
If we cannot aspire to the significant degree of social
and economic equality that is required for formalism
to be compatible with the achievement of substantive
justice, then formalism should be abandoned so that the
Law School Admission Council
3 3 3 3 -19-
3
3. The passage most strongly supports which one of the 5. The passage indicates that which one of the following
following statements about a formalist approach to is a characteristic of a body of law that delivers merely
the law? procedural justice?
(A) It tends to be too detailed in its language and is (A) It cannot be relied upon to deliver results that are
not sufficiently reliant on broad principles. recognizably fair and equitable.
(B) It may satisfy the demands of objective reason (B) It derives from subjective experience as well as
and yet fail to connect to social reality. objective reason.
(C) It produces substantive justice in individual cases (C) It adapts too readily to social reality.
if it is applied neutrally and without prejudice. (D) It takes into account the social and economic
(D) Its tension with substantive justice arises circumstances of those who come before
from their differing approaches to deductive the law.
reasoning. (E) It is based on a common understanding of
(E) It can be reconciled with substantive justice by fairness and justice.
formalizing a recognition of social inequalities.
4. The author’s primary purpose in mentioning the fact that
someone must lose in an adversarial system of law (third
sentence of the second paragraph) is to
(A) emphasize the systemic nature of the tension GO ON TO THE NEXT PAGE.
between formalism and substantive justice
(B) imply that, invariably, someone will view the
outcome of a legal proceeding as unfair
(C) suggest that the flaws of formalism may be absent
from some systems of law
(D) stress that substantive justice cannot be expected
to produce outcomes that favor every participant
(E) foreshadow the author’s conclusion about the
ultimate irreconcilability of formalism and
substantive justice
Law School Admission Council
3 -20-
3 3 3 3
Passage A is adapted from a book review by physicist results are unequivocal: psychic power is a chimera.
Freeman Dyson. Passage B is adapted from a response Dyson recognizes that his belief in the possibility of
to the review. paranormal phenomena conflicts with his scientific
views, but he responds by invoking the principle of
complementarity. Sorry, but the principle does not apply
Passage A
to the paranormal. Either people can read other people’s
There are two extreme views concerning the role
minds or they cannot. Science has demonstrated that
of science in human understanding. The reductionist
they cannot. And being a holist instead of a reductionist
view holds that all kinds of knowledge, from physics to
or reading about weird things that happen to people
ethics, can be reduced to science. The traditional view
does not change this scientific fact.
holds that science is one of many independent sources
of knowledge. Most people hold views between these 6. Both passages are concerned with answering which one
two extremes. Skeptics of paranormal phenomena are of the following questions?
generally near the reductionist extreme, while I am near
the traditional extreme. (A) Does experiencing apparent paranormal
The question of the limits of science is closely phenomena lead to stress?
connected to the possible existence of paranormal (B) Is it possible that paranormal phenomena exist?
phenomena. Scientific attempts to study extrasensory (C) Do anecdotes have any place in scientific
perception and telepathy have failed. Skeptics conclude inquiry?
from this that paranormal phenomena do not exist. (D) Is the principle of complementarity valid outside
I do not accept this conclusion because I am not a quantum physics?
reductionist. Paranormal phenomena may exist but be (E) Are there kinds of knowledge that have not been
inaccessible to scientific investigation. This is just a successfully reduced to science?
hypothesis, but one that I find tenable and plausible.
This hypothesis is supported by abundant 7. It can be inferred that the author of passage A would
evidence (stories of ordinary people who apparently be most likely to consider which one of the following
possess paranormal abilities) collected by the Society assertions made in passage B to misrepresent his views?
for Psychical Research and similar organizations. (A) The author of passage A is convinced that
This evidence is anecdotal rather than scientific, under certain conditions some people exhibit
since it cannot be reproduced under controlled paranormal powers.
conditions. But the organizations have conscientiously (B) The author of passage A believes that there
interviewed eyewitnesses right after the events and is anecdotal evidence for the existence of
carefully documented the stories. These stories make paranormal phenomena.
it clear that if paranormal events occur, they occur (C) The author of passage A denies that he holds the
only when people are under stress and experiencing reductionist view of the role of science in human
strong emotion. This would explain why paranormal understanding.
phenomena are not observable in well-controlled (D) The author of passage A tries to resolve the
scientific experiments. Strong emotion and stress are conflict between science and the paranormal by
incompatible with scientific procedures. invoking the principle of complementarity.
Paranormal phenomena and the scientific (E) The author of passage A believes that paranormal
method may be complementary. “Complementary” powers are not observable in scientific
is a technical term in quantum physics, meaning that experiments.
two descriptions of nature are both valid but cannot
be observed simultaneously. The classic example of 8. In the first sentence of passage B, the author says that
complementarity is the dual nature of light. Light paranormal powers “mysteriously disappear” when
behaves as a wave in one experiment and as particles in people are placed in controlled scientific conditions
another, but we cannot see both in the same experiment. primarily in order to
Passage B (A) present a counterexample to reductionism
Freeman Dyson makes a ridiculous plea for (B) introduce the author’s view of the role of science
openness to the paranormal because he is not a in human understanding
reductionist and because anecdotal evidence convinces (C) state the author’s main point of disagreement
him that under certain conditions (e.g., stress), some with passage A
people exhibit paranormal powers, unless they are (D) ridicule a view expressed in passage A
placed in controlled scientific conditions, in which (E) convey the results of scientific investigations of
case the powers mysteriously disappear. A scientist the paranormal
of Dyson’s caliber should know that anecdotes do not
make a science. The only way to find out if anecdotes
represent real phenomena is controlled experimental
tests. Paranormal phenomena have repeatedly been
subjected to rigorous scientific experiments, and the GO ON TO THE NEXT PAGE.
Law School Admission Council
3 3 3 3 -21-
3
9. Which one of the following principles underlies the 12. Which one of the following statements most accurately
argument in passage B but would be rejected by the describes the relationship between passage B and the
author of passage A? assertion in passage A that “skeptics of paranormal
phenomena are generally near the reductionist extreme”
(A) There are principles of quantum physics that
(last sentence of the first paragraph)?
cannot be applied to studies of human behavior.
(B) Either all knowledge can be reduced to science, (A) Passage B attempts to disprove that assertion.
or science is in no way privileged over other (B) Passage B dismisses that assertion as not worthy
sources of knowledge. of a response.
(C) Scientific facts are valid independently of any (C) Passage B serves as an example of that assertion.
views that people may have about the role of (D) Passage B examines the relationship of that
science in human understanding. assertion to the principle of complementarity.
(D) If the existence of an alleged phenomenon cannot (E) Passage B attempts to show that that assertion
be demonstrated in controlled experimental is incompatible with other assertions made in
tests, then the phenomenon does not exist. passage A.
(E) The support that anecdotes provide for a
scientific theory can never be as strong as the 13. It can be inferred from the passages that the author
support provided by controlled experimental of passage B would be more likely than the author
tests. of passage A to accept which one of the following
statements?
10. The primary purposes of the two passages are related in
(A) There may be real phenomena that can never be
which one of the following ways?
discovered by scientific procedures.
(A) Passage A argues for a general view, and (B) There are numerous reports citing ordinary
passage B presents a specific counterexample to people who apparently demonstrate paranormal
that view. abilities.
(B) Passage A proposes a wide-ranging new theory, (C) Light behaves as a wave in some experiments
and passage B takes issue with some details of and as particles in others.
that theory. (D) Scientific investigation has provided no evidence
(C) Passage A argues that something is possible, and that extrasensory perception and telepathy
passage B tries to refute that argument. are real.
(D) Passage A questions a mainstream view, and (E) The same general methods that are used in
passage B defends that view against the charges physics are appropriate to the study of all human
made in passage A. behavior.
(E) Passage A argues that a theory is self-
contradictory, and passage B argues that the
contradiction is only apparent.
11. Passage A, but not passage B, refers to
(A) scientific experiments
(B) anecdotal evidence GO ON TO THE NEXT PAGE.
(C) stress
(D) reductionism
(E) the dual nature of light
Law School Admission Council
3 -22-
3 3 3 3
In typical accounts of the beginnings of bebop— 14. Which one of the following most accurately states the
the first “modern” jazz style, which was originated main point of the passage?
in the 1940s by Dizzy Gillespie, Charlie Parker, and
(A) Historical inquiry into the origins of bebop has
Thelonious Monk, among others—commercialism
tended to overemphasize the commercial causes
plays an important, though indirect, role. By the early
of swing music’s artistic decline.
1940s, according to these histories, jazz had reached an
(B) Typical accounts of the origins of bebop
impasse. The reigning jazz style, swing, had become
misrepresent the relationship of bebop’s
“threadbare,” a “harmonic and melodic blind alley,”
originators to mass-market capitalism.
a formulaic popular music undergoing “death by
(C) Bebop arose as a reaction to the failure of swing
entropy,” a “billion-dollar rut.”
musicians to extend jazz’s rhythmic, harmonic,
These metaphors, sampled from various writings
and melodic language in directions plainly
on jazz, echo the “crisis theory” of twentieth-century
indicated by the music itself.
European classical music. Classical music history
(D) Commercial relations permeate all genres of
textbooks commonly impute the eruptions of modernity
musical entertainment, including bebop.
in the early 1900s to classical music’s stubborn failure
(E) Bebop’s originators did not see bebop as being
to move beyond the language of tonality worn out from
fundamentally different from swing music.
overuse in the nineteenth century. Something similar is
implied about jazz in the early 1940s. Musicians’ failure 15. According to the author’s argument, at least some of the
to extend jazz’s rhythmic, harmonic, and melodic originators of bebop were motivated by a desire to
language in directions plainly indicated by the music
itself built up pressure resulting in the eruption of a new (A) attain artistic autonomy
musical modernism. (B) overthrow the musical dominance of swing music
But phrases like “billion-dollar rut” clearly (C) strip jazz of its associations with entertainment
suggest that these writers believe that the real culprit is (D) escape the commercial influences present in
commercialism—the commingling of art and commerce swing music
that had for a time allowed swing to become both an (E) secure financial rewards greater than those
authentic jazz expression and a national fad. Even available to swing musicians
after swing had run its course, the theory goes, the
machinery of the popular music industry continued to 16. The primary purpose of the reference to eruptions of
prop up the “threadbare” idiom, seducing musicians modernity in classical music in the early 1900s (second
into going through the motions long after they had sentence of the second paragraph) is to
any legitimate artistic reason to do so. In other words, (A) provide evidence that undermines the typical
mass-market capitalism was a logjam in the path of accounts of the origins of bebop by suggesting
musical evolution that could be removed only by that factors other than commercialism were
explosive force. Bebop provided that force. In this at play
version of jazz history there is an implicit teleology to (B) outline a theory of classical music history that the
the progression from early jazz through swing to bebop: author claims is parallel to the typical accounts
the gradual shedding of jazz’s associations with dance, of the origins of bebop
popular song, and entertainment. Bebop is the logical (C) suggest that typical accounts of the origins of
culmination of this process—in it jazz became “art,” bebop are based on an inaccurate understanding
declaring its autonomy by severing forever its ties to of the history of twentieth-century music in
commerce. general
This insistence that bebop is anticommercial may (D) describe a movement in classical music that was
suit the needs of contemporary jazz discourse, but it part of the impetus behind the transition from
is a poor basis for historical inquiry. It idealizes the swing to bebop in jazz
circumstances of artistic creation and represses the (E) provide an example of a modernist movement
unpleasant reality that commercial relations permeate in classical music that was motivated at least in
all realms of musical entertainment. For the musicians part by commercial considerations
who originated bebop, mass-market capitalism was not
a prison from which the true artist was duty-bound to
escape, but a system of transactions defining music as a
profession, thereby making their achievements possible.
By 1945, Parker, Gillespie, and Monk had indeed willed
a new musical subculture into being. But they were not
trying to disengage from the “commercial” music world GO ON TO THE NEXT PAGE.
so much as to find a new point of engagement with it—
one that would grant them a measure of autonomy and
recognition.
Law School Admission Council
3 3 3 3 -23-
3
17. The author of the passage would be most likely to agree 20. It can be inferred from the passage that the authors
with which one of the following statements about the of typical accounts of the origins of bebop would be
originators of bebop? most likely to agree with which one of the following
statements about swing musicians in the 1940s?
(A) Their music was promoted by the music industry
with the same vigor as swing music had been (A) They continued to believe that their music was
earlier. innovative even though artistic developments in
(B) They repudiated the notion accepted by swing jazz had rendered swing outdated.
musicians that jazz was a form of popular (B) They resented critics’ insinuations that their
entertainment. music was merely entertainment.
(C) They regarded themselves as professionals and (C) They recorded and performed music in a style
accepted commercialism as a central element of that many of them no longer found to be
their profession. artistically compelling.
(D) They were better off financially than they would (D) They believed they had a responsibility to
have been had they played only swing music. preserve the great traditions established by
(E) They believed that bebop would appeal to as earlier generations of jazz musicians.
wide an audience as swing did. (E) They sought unsuccessfully to liberate their
music from the pressures of commercialism.
18. It can be inferred from the passage that the proponents
of the typical accounts of the origins of bebop would be 21. Which one of the following principles does the author
most likely to believe which one of the following? use in analyzing typical accounts of the origins of bebop?
(A) The lack of innovation in classical music in the (A) Artistic progress in music is generally the result
early 1900s was due largely to commercialism. of commercial pressures.
(B) Swing music came to enjoy immense commercial (B) New movements in music typically begin with
success primarily because it was not the rejection of the fundamental principles of the
aesthetically adventurous. reigning musical style.
(C) The insistence that bebop was anticommercial (C) Music historians should rely primarily on
in its origins serves the needs of today’s jazz musicians’ first-hand accounts in analyzing
critics rather than the needs of genuine historical significant developments in music.
inquiry. (D) The turns of phrase employed by historians
(D) Swing contained the seeds of innovations in can legitimately be analyzed to uncover the
musical language that, because of commercial historians’ assumptions.
pressures, were left undeveloped by swing (E) Music historians must take care not to let their
musicians. aesthetic preferences influence their historical
(E) The originators of bebop embraced jazz’s analyses.
associations with dance, popular song, and
entertainment.
19. According to the passage, which one of the following is
true of typical accounts of the origins of bebop?
(A) They assert that bebop was originated by GO ON TO THE NEXT PAGE.
musicians seeking commercial success.
(B) They represent bebop as an outgrowth of the
modernist movement in classical music.
(C) They identify bebop as the first jazz movement
that did not have strong commercial appeal.
(D) They overly idealize the realities of artistic
creation.
(E) They were themselves shaped by commercial
pressures.
Law School Admission Council
3 -24-
3 3 3 3
Organic chemist Larry Koskan’s inspiration fertilizers. If Koskan’s company can show continued
arrived in the mid-1980s, when he read a report by success, it may yet help enliven an environmental
marine biologists describing how oyster shells grow. It chemistry sector saddled with a reputation for
was known that oysters secrete calcium carbonate as an ineffectiveness and high cost.
essential constituent of their hardened exteriors, but the
biologists discovered that they also produce a special 22. Which one of the following most accurately expresses the
protein-based agent that molds the mineral into their main point of the passage?
shells’ characteristic shape. (A) Polyaspartate was once thought to be the
At the time, Koskan was studying the properties inevitable successor to polyacrylates, but
of water-soluble polyacrylates. Among other things, technical shortcomings restrict its usefulness to
these widely used additives help to stem the buildup applications in specialized niche markets.
of damaging mineral-scale deposits (carbonate and (B) The success of Koskan’s company in the
sulphate compounds) on the surfaces of industrial development of polyaspartate for industrial
water-treatment equipment. What Koskan realized was uses may improve the reputation of the
that the agent produced by the oysters—polyaspartate— environmental chemical industry.
inhibits the formation of calcium carbonate and could (C) Polyaspartate, which Koskan researched and
also therefore control scale. Polyaspartate mimics the developed as an environmentally friendly
scale-inhibiting activity of polyacrylates because it has alternative to polyacrylates, shows promise for
a similar chemical structure. But because the backbone commercial applications but also faces barriers.
of polyaspartate is made of peptides (chains of amino (D) Koskan’s work with polyaspartate has shown that
acids) rather than the hydrocarbon compounds that environmentally friendly alternatives to many
constitute polyacrylates’ backbone, it is subject to industrial chemicals can be found by studying
bacterial action (i.e., it is biodegradable). plants and animals.
Polyacrylates, inexpensive and versatile (E) Although polyaspartate is superior to existing
chemicals, are easy to manufacture and process. In products, it will probably fail in the marketplace
laundry detergents, they act as dispersants that keep dirt because of hurdles that stand in the way of its
suspended in the wash water. As a recent alternative being economically competitive.
to phosphates, which, via wastewater, pollute surface
waters, some half a billion pounds of polyacrylates are 23. Which one of the following scenarios would conform
used in detergents worldwide every year. Additionally, most closely to the assessment given in the final sentence
some polyacrylates have chemical configurations with of the passage?
a tremendous affinity for water, making them ideal in
superabsorbent materials for disposable products like (A) Koskan’s company produces a carefully
baby diapers, which currently account for around 2 designed marketing video that demonstrates the
billion pounds in annual production of polyacrylates. effectiveness of polyaspartate in eliminating
The trouble is, polyacrylates last virtually forever, and scale in both oil production equipment and
these vast quantities of the highly stable substance are water treatment equipment.
being discarded in landfills. (B) Koskan’s company comes under increased
With requests by consumer products companies market competition from leading chemical
for environmentally friendly products growing companies that are developing alternatives to
rapidly, Koskan started his own small company and polyaspartate to reduce scale in offshore oil
began researching cost-effective ways of producing equipment.
polyaspartate for industrial use. Soon, leading (C) Using revenues from sales of polyaspartate to the
chemical companies also began researching the oil and farming industries, Koskan’s company
issue, with the consensus being that polyaspartate is able to develop low-cost ways of producing
was the most suitable replacement for polyacrylates. large volumes of polyaspartate for general use.
Unfortunately, as with many emerging technologies (D) Koskan’s company performs studies
that lack a supporting infrastructure, polyaspartate’s demonstrating that polyacrylates are
price was estimated to be four to five times that accumulating in landfills at a rate that is
of the high-volume-production alternative. Given increasing steadily.
this, the consumer products industry lost interest in (E) Koskan’s company discovers that oysters secrete
the technology, and the chemical industry shelved a highly specialized form of polyaspartate that is
it. Koskan’s company, however, believing that difficult to isolate in laboratory settings.
polyaspartate could nonetheless be market-competitive
on its own merits, decided to persevere and focus on
specialty application niches. Polyaspartate has since
shown unparalleled success at eliminating scale in
offshore oil production equipment and is being used by
the farming industry to keep chemical fertilizers in the GO ON TO THE NEXT PAGE.
soil longer, reducing pollution of groundwater by the
Law School Admission Council
3 3 3 3 -25-
3
24. Which one of the following most accurately captures the 27. The passage most strongly suggests that which one of the
meaning of the phrase “on its own merits” as it is used in following is true of both polyaspartate and polyacrylates?
the third-to-last sentence of the passage?
(A) They perform identically within any industrial
(A) without funding from government agencies application.
(B) without reference to its chemical properties (B) They were both discovered as a result of research
(C) once its newest industrial uses are more widely performed by marine biologists.
publicized (C) The chemical industry was aware of their
(D) not merely as a general substitute for existence long before industrial applications for
polyacrylates either one were developed.
(E) because of its unsurpassed ability to reduce scale (D) They both have been used to reduce water
pollution.
25. It can most reasonably be inferred from the passage (E) At the time they were first brought to market,
that the author believes that the failure of leading leading chemical companies were skeptical
chemical companies to fully embrace polyaspartate as a about their commercial success.
replacement for polyacrylates
(A) was a result of fears about the potentially harmful
consequences to human health of using a new
and untested material
(B) was not primarily a result of considerations
involving costs and profits
(C) was not due to a widely accepted belief that
polyaspartate is ineffectual
(D) was a result of the relative scarcity of the raw
material from which polyaspartate is made
(E) was due to the belief that consumers would not
trust products containing polyaspartate
26. The primary purpose of the passage is to
(A) describe and analyze one attempt to develop
and market a new, promising alternative to an
existing product
(B) demonstrate the mistakes made in one attempt to
develop and market a new, promising alternative
to an existing product
(C) illustrate the kinds of problems that are likely
to be encountered in attempts to market a new,
promising alternative to an existing product
(D) comment on the state of affairs in an industry
that tends to overlook environmentally friendly
innovation in favor of maintaining strong profits
(E) explain the chemical processes that make a
particular new product a promising alternative
to an existing product
S T O P
IF YOU FINISH BEFORE TIME IS CALLED, YOU MAY CHECK YOUR WORK ON THIS SECTION ONLY.
DO NOT WORK ON ANY OTHER SECTION IN THE TEST.
Law School Admission Council
4 -26-
4 4 4 4 4
SECTION IV
Time—35 minutes
26 Questions
Directions: Each question in this section is based on the reasoning presented in a brief passage. In answering the questions, you
should not make assumptions that are by commonsense standards implausible, superfluous, or incompatible with the passage. For
some questions, more than one of the choices could conceivably answer the question. However, you are to choose the best
answer; that is, choose the response that most accurately and completely answers the question and mark that response on your
answer sheet.
1. Montoya: Many industrial chemical processes that 3. James: The world is increasingly divided between the
currently use organic solvents could use ionic computer literate and the computer illiterate.
liquids instead. Ionic liquids are less hazardous The economic gap between rich and poor is
to workers and generate less air pollution. partly responsible for causing this division, but
Moreover, some reactions occur at a faster rate or the economic gap will widen because of the
yield smaller quantities of unwanted by-products increasing importance of computer literacy.
when ionic liquids are used. So chemical Ariel: I disagree. Because of businesses’ increasing need
companies should begin using ionic liquids for computer-literate employees, companies
for many reactions that currently use organic will have to train nearly all workers in computer
solvents. skills. This will lessen the division between the
Peterson: Ionic liquids cost many times as much as computer literate and the computer illiterate. So
organic solvents, so they are currently not whatever might cause the economic gap between
practical for the chemical industry. rich and poor to widen in the future, it won’t be
the importance of computer literacy.
Of the following, which one, if true, is the strongest
counter Montoya could make to Peterson’s argument? James and Ariel disagree over whether
(A) The cost of organic solvents is only one of many (A) the economic gap between rich and poor will
expenses involved in industrial production of widen
chemicals. (B) the economic gap between rich and poor is now
(B) New methods for removing by-products partly responsible for causing the division
of chemical reactions have recently been between the computer literate and the computer
developed. illiterate
(C) The chemical industry has historically been quick (C) businesses’ need for computer-literate employees
to adopt new techniques that increase the rate at will increase
which reactions occur. (D) the economic gap between rich and poor will
(D) Ionic liquids can be reused many times, whereas widen as a result of the increasing importance of
organic solvents can be used only once. computer literacy
(E) For the sake of public relations, companies will (E) companies will have to train their employees in
sometimes use a more environmentally friendly computer skills
process even if it is slightly more expensive.
2. The relationship between money and the things it allows
one to purchase is like that between a tool and the tasks it
enables its user to accomplish. Therefore, since tools are
useless if there is no task that needs to be done, _______.
GO ON TO THE NEXT PAGE.
Which one of the following most logically completes the
argument?
(A) money should be thought of as no more useful
than a tool
(B) any tool that enables its user to accomplish a task
is valuable
(C) money is valueless in a world where nothing is
for sale
(D) money should be regarded as a tool rather than a
commodity
(E) the value of money derives from the tasks it
allows one to accomplish
Law School Admission Council
4 4 4 4 4 -27-
4
4. Overcrowding inclines human beings toward aggressive, 6. Anthropologist: During the last ice age, nomadic
competitive behavior. Just consider the behavior of communities probably needed at least 15 or 20
normally considerate people driving in rush-hour traffic members to survive, and they were generally
or attending crowded sporting events. This shows that, as not much larger than this. Ice-age nomads are
world population continues to grow, aggressive behavior, commonly portrayed as primarily big-game
including war and crime, will become more frequent and hunters, but most of their food must have in fact
severe. come from other sources, such as small game
and plants. Hunting large animals is a dangerous
Each of the following, if true, strengthens the argument
activity that would have risked the lives of
EXCEPT:
several members of the community.
(A) The frequency and severity of overcrowding in
Which one of the following most accurately describes
the past have been roughly proportional to the
the role played in the argumentation by the statement
size of world population.
that ice-age nomads are commonly portrayed as
(B) The frequency and severity of war and crime
primarily big-game hunters?
have generally increased in the past century,
and world population also increased during that (A) It is a premise used as support for the overall
period. conclusion of the anthropologist’s argument.
(C) Some forms of aggressive, competitive behavior, (B) It is a clarification of one of the premises of the
including war, tend to occur even in situations in anthropologist’s argument.
which human beings are not overcrowded. (C) It is an objection that the anthropologist raises
(D) With respect to its effect on behavior, rush-hour against an opposing theory.
traffic is an example of a situation in which (D) It is the overall conclusion of the anthropologist’s
human beings are overcrowded. argument.
(E) Most mammals inhabiting a given region exhibit (E) It describes a claim that the anthropologist
increasingly aggressive, competitive behavior as attempts to refute.
their population density in that region increases.
7. A truly visual art form—for example, painting—is one in
5. Gahagan’s Greenstore sells a large volume of plants. The which time plays no essential role. Though it takes time
vast majority are sold wholesale to commercial buyers, to look at a painting, there is no fixed order in which one
most of them to landscape contractors. Gahagan’s also must look at its parts, and no fixed amount of time one
sells gardening implements, most of which are purchased must spend examining it. In contrast, most art forms,
retail by home gardeners. such as poetry and music, are essentially temporal; that
is, they require performance, which means they must
Which one of the following is most strongly supported
be experienced in a fixed order and over a roughly fixed
by the information above?
amount of time. Poetry, for instance, though often written
(A) Gahagan’s makes a larger profit from the sale of down and thus seemingly a visual art, actually must
plants than from the sale of all other products be performed, even if the performance is only a silent
combined. reading to oneself.
(B) Most of those who make regular wholesale
If the statements above are true, then on the basis of
purchases from Gahagan’s have never purchased
them which one of the following must be true?
gardening implements from Gahagan’s.
(C) There are more commercial buyers than home (A) Truly visual art forms do not essentially involve
gardeners among Gahagan’s regular customers. performance.
(D) Gahagan’s sells a few gardening implements (B) Poetry is less like music than it is like painting.
wholesale to landscape contractors. (C) Spatiality and temporality are mutually exclusive
(E) A plant purchased from Gahagan’s is more components of art forms.
likely to have been purchased by a landscape (D) Art forms that must be examined for an extended
contractor than by a home gardener. period of time in order to be understood are
essentially temporal.
(E) Anything capable of being performed is either
musical or poetic, or both musical and poetic.
GO ON TO THE NEXT PAGE.
Law School Admission Council
4 -28-
4 4 4 4 4
8. Actors generally learn their lines by focusing on the 10. While the population of city X is approximately one-half
meanings of the words, the motivations of the characters that of city Y, the number of city X residents who are
uttering them, and the physical and emotional dimensions patients in hospitals is only one-fourth that of the number
of their own performances. It seems likely that memory of city Y residents who are patients in hospitals.
can be enhanced by factors such as emotion, action, and
Each of the following, if true, contributes to an
context, so this approach is probably more effective than
explanation of the difference in the number of residents
mere rote memorization.
who are patients in hospitals EXCEPT:
Each of the following, if true, adds support for the
(A) Preventive health programs are more prevalent in
conclusion drawn above EXCEPT:
city X than in city Y.
(A) Test subjects are best able to remember items (B) The hospitals in city X are noted as leaders
on a shopping list when they are also told how in employing outpatient treatment wherever
those items will be used. possible.
(B) The actors who are able to deliver the greatest (C) The drinking water of city Y has dangerously
amount of dialogue most effortlessly are those high levels of pollutants, whereas this is not the
who have spent the most time learning their case for city X.
lines. (D) The hospitals in city Y are of very high quality,
(C) Actors are more readily able to remember lines and residents of city X are often sent there for
learned while making an appropriate motion— treatment.
for example, walking across a stage—than lines (E) The lifestyle in city X is significantly less
learned without an accompanying action. stressful than the lifestyle in city Y.
(D) People who try to memorize information
by imagining that they are conveying it to 11. A law is futile unless most of the parties subject
somebody else who needs the information show to it abide by it willingly. Thus, there can be no
higher retention than those who try to memorize comprehensive international solution to the problem of
the material by rote. pollution, since few nations would willingly give up their
(E) People with no acting experience are able to power over their own industries.
memorize and deliver lines from a play more Which one of the following is an assumption required by
accurately after they have read and understood the argument?
the entire play.
(A) A comprehensive solution to the problems of
9. Journalists often claim that their investigation of the pollution would require major changes in
private lives of political leaders is an effort to improve international law.
society by forcing the powerful to conform to the same (B) Some nations would be required to give up some
standards of conduct as the less powerful. In reality, of their power over their own industries as part
however, the tactic is detrimental to society. It makes of any comprehensive solution to the problem of
public figures more concerned with mere appearances, pollution.
and makes everyone else cynical about the character of (C) If most nations were willing to abide by
their leaders. international law, then there could be a
comprehensive solution to the problem of
The claim that journalistic investigation of the private
pollution.
lives of political leaders is an effort to improve society
(D) The problems created by pollution affect most
plays which one of the following roles in the argument?
of the world’s nations, but the pollution itself
(A) It is a claim that the argument attempts to refute. is produced by the industries of relatively few
(B) It mentions a justification that is sometimes nations.
offered for a practice that, the argument (E) Most of the current international laws concerning
concludes, has undesirable consequences. pollution do not require nations to give up any
(C) It is cited as evidence often given for an assertion of their power over their own industries.
that the argument concludes is false.
(D) It describes a phenomenon that, according to the
argument, is much less damaging to society than
journalists often assume.
(E) It gives an example of a phenomenon that the
argument contends has very different effects GO ON TO THE NEXT PAGE.
from those it is generally assumed by everyone
to have.
Law School Admission Council
4 4 4 4 4 -29-
4
12. Ecologist: El Niño, a global weather phenomenon that 14. Science writer: Scientists often accept a scientific
occurs once every several years, is expected to argument largely for social reasons, such as the
become more frequent in coming decades due to argument’s acceptance by other scientists or the
the global warming caused by air pollution. In prestige of the scientist making the argument.
region T, El Niño causes heavy winter rainfall. But this is not at all to the sciences’ detriment,
Since rodent populations typically increase for similar social factors strongly influence every
during long periods of sustained rain, it is likely human endeavor.
that average rodent populations in region T will
The science writer’s argument is most vulnerable to
also increase in coming decades.
criticism on the grounds that it
Which one of the following, if true, would most weaken
(A) takes for granted that the only legitimate reasons
the ecologist’s argument?
for accepting an argument are social reasons
(A) In region T, there is typically much less rainfall in (B) takes for granted that whether a behavior
summer than there is in winter. obstructs the achievement of a particular
(B) Rodent populations in region T often diminish endeavor’s goals depends on the prevalence of
during long periods in which there are no heavy comparable behaviors in other endeavors
rains. (C) fails to consider the possibility that scientists who
(C) In many regions that, on average, experience accept an argument largely for social reasons
substantially more winter rainfall than also generally consider relevant scientific
region T does, average rodent populations are evidence before accepting the argument
considerably lower than they are in region T. (D) fails to consider the possibility that many
(D) In region T, winters marked by relatively high scientists are unaware that they usually accept
rainfall have usually not been marked by long scientific arguments largely for social reasons
periods of sustained rain. (E) takes for granted that a behavior that is not
(E) The global warming caused by air pollution detrimental to the sciences will not be
produces a number of effects, other than the detrimental to any other human endeavor
increase in the frequency of El Niño, that could
affect rodent populations. 15. Historian: Scholars writing histories of an era’s business
practices must, of course, analyze the practices
13. If the natural history museum stays within this year’s and strategies employed by firms of that era.
budget, it will be unable to stay within next year’s But historians probably study successful firms
budget, for renovating next year will make the museum’s more frequently than they do unsuccessful firms.
expenditures exceed next year’s very tight budget. After Therefore, it is reasonable to think that business
all, the museum will have to renovate next year if it histories overestimate the successes of past
does not do so this year, because work from previous businesses.
renovations is deteriorating rapidly.
Which one of the following would, if true, most support
The argument’s conclusion can be properly inferred if the historian’s argument?
which one of the following is assumed?
(A) The specific factors that cause businesses to do
(A) The museum will stay within this year’s budget. poorly are often not inferable from historical
(B) This year’s budget is less than next year’s budget. records of business activity.
(C) The museum will not renovate next year. (B) Those who study past first-person accounts of
(D) The museum will exceed this year’s budget if it business strategies inevitably approach them
renovates this year. with certain present-day cultural assumptions.
(E) The museum will stay within this year’s budget if (C) The many public legal documents that firms
it does not renovate this year. have had to file provide a record that is more
objective than that provided by the firms’ own
internal documents.
(D) The records of businesses that have gone
bankrupt are destroyed more frequently than the
records of other businesses.
(E) Scholars who study businesses of the past are
usually trained in the effective techniques of
business administration.
GO ON TO THE NEXT PAGE.
Law School Admission Council
4 -30-
4 4 4 4 4
16. Manager: I plan to put together a successful marketing 18. Land developer: In a certain nation, stringent regulations
team by choosing highly skilled, independent prevent private landowners from building on
workers who prefer not to work as part of a their land if any endangered species is present
tightly knit group. This strategy is bound to on it. These regulations make the presence of
succeed because recent research shows that when endangered species a severe financial liability
a common purpose is shared, the flexibility of for many landowners and thereby discourage
loosely bound groups enables them to function the landowners from protecting the endangered
better than tightly knit ones. species on their land. Therefore, endangered
species would very likely not be harmed by
The reasoning in the manager’s argument is most
removal of the regulations on land development.
vulnerable to criticism on the grounds that the argument
The reasoning in the land developer’s argument is most
(A) presumes that factors that are collectively
vulnerable to criticism on which one of the following
sufficient for success are also individually
grounds?
necessary for success
(B) relies on research that has no apparent relevance (A) It confuses a condition whose presence would
to the manager’s proposed plan be required in order for a certain result to be
(C) takes for granted that flexible teams always produced with a condition whose presence
function significantly better than inflexible would be required to prevent that result from
teams occurring.
(D) overlooks the possibility that people who prefer (B) It justifies a claim containing a value judgment
not to work in tightly knit groups may have solely on the basis of factual claims that do not
difficulty committing to a common purpose in themselves entail that value judgment.
(E) confuses the goals that the manager is seeking (C) It unjustifiably overlooks the possibility that
to achieve with the goals that must be held in even if certain factors tend to produce a given
common by the members of the marketing team effect, they may be likely to produce stronger
countervailing effects as well.
17. Different cultures have independently reached the (D) It fails to take into account the possibility that
conclusion that ingesting honey daily promotes vigor policies that are not in the interest of land
and longevity. Thus, unless strong evidence emerges developers may well be in the interest of
contradicting this conclusion, we should assume that it landowners.
is true. (E) It fails to account for potential reactions from
private landowners who do not have any
The pattern of reasoning in which one of the following is
endangered species on their land.
most similar to that in the argument above?
(A) Three engineers have inspected the bridge, and
each has independently concluded that it is safe.
So unless good evidence emerges that the bridge
is unsafe, it should be considered safe.
(B) Numerous studies have concluded that most GO ON TO THE NEXT PAGE.
people benefit from weight-training exercises.
Thus, since no other form of exercise has been
shown to be more beneficial, most people should
be encouraged to exercise with weights.
(C) The members of an independent group of
scientists have concluded that human activities
over the last century have disrupted global
weather patterns. Thus, unless these scientists
are working from questionable assumptions, we
should conclude that their assessment is correct.
(D) Three theater critics, none of whom had any
influence on the others, said that Tales in Tokyo
was not worth the price of admission. Thus,
unless each of these critics happened to attend
the play on the same night, it is unlikely that the
play will be successful.
(E) Several different physicians examined Janet, and
none could agree on a diagnosis. Therefore,
unless new evidence shows that Janet is
unhealthy, she should instead assume that she is
healthy.
Law School Admission Council
4 4 4 4 4 -31-
4
19. I promised Bernie that I would answer his question. 21. Company policy: Eligible employees will be granted up
He asked me whether his project had been approved. I to six weeks of paid parental leave following
misunderstood and thought he had asked whether I had the birth or adoption of a child. To be eligible,
approved it. I said, truthfully, “I would approve it if I an employee must apply for the benefit at least
could, but I don’t have the authority to do so—that is up three months prior to the beginning of the leave
to Dorothy, and she hasn’t yet made a decision.” Thus, I period and must have at least one year of full-
fulfilled my promise to Bernie. time employment with the company at the time of
application.
Which one of the following, if assumed, enables the
conclusion above to be properly drawn? Which one of the following judgments most closely
conforms to the company policy?
(A) All there is to answering a question is giving the
questioner the information requested. (A) Mac is a full-time employee who has been with
(B) No person can be held accountable for another the company for exactly one year. He just
person’s decisions. adopted a child, and he applied for parental
(C) The person responsible for a decision is the one leave three months ago, so Mac should be
who should explain the decision. granted the leave.
(D) One need not fulfill a promise in order to do the (B) Sara has worked full-time for the company for
best that could be done to fulfill it. three years, and she applied for parental leave
(E) Making a promise always obliges a person to four months before the recent birth of her child.
keep it. However, she should not be granted the leave
because she took a leave of absence within the
20. Azalea bushes flourish in acidic soil. Clay soil, while past year.
rarely acidic, can be mixed with other types of soil to (C) Hal, a second-year, full-time employee, applied
create a soil that is acidic. Hence, azalea bushes can be for parental leave three months ago. He wishes
made to flourish in clay soil. to stay at home for only two weeks following
Which one of the following is an assumption on which the impending birth of his child, so the company
the argument depends? should not grant him parental leave.
(D) Dudley has worked part-time for the company
(A) Mixing an acidic soil with another type of soil for the past seven years. He applied for parental
always results in an acidic soil. leave over three months before the birth of his
(B) Clay soil that is mixed with certain other types of child, so it should be granted to him.
soil is still clay soil. (E) Maria has worked full-time for the company for
(C) Whenever clay soil is mixed with other types of the past two years and applied for parental leave
soil, an acidic soil results. six months prior to her adoption of a child, so
(D) Some plants do not flourish in acidic soil. she should be granted parental leave.
(E) Azalea bushes do as well in treated clay soils as
in naturally acidic soils.
GO ON TO THE NEXT PAGE.
Law School Admission Council
4 -32-
4 4 4 4 4
22. An effective acting performance does not call the 24. A movie studio’s script readers discard all proposed
audience’s attention to the fact that it is a performance. scripts that are not submitted by agents, and they discard
That would make it more difficult for the audience to all scripts that are incorrectly formatted. So a script that
empathize with the character the actor is playing, and is submitted by an agent and formatted correctly will not
effective dramatic performances do not detract from the be discarded by the studio’s script readers.
audience’s appreciation of a play.
Which one of the following exhibits a flawed pattern of
The argument’s conclusion follows logically from the reasoning that most closely parallels that in the argument
premises if which one of the following is assumed? above?
(A) An audience will not completely appreciate a (A) To be accepted, entries in the essay contest must
play unless all of the acting performances in the be postmarked by the deadline and accompanied
play are effective. by a completed entry form. March 21 was the
(B) As long as an acting performance does not deadline. Thus, an entry that is accompanied
call the audience’s attention to the fact that it by a completed entry form but postmarked
is a performance, it will not detract from the March 22 will not be accepted.
audience’s appreciation of a play. (B) If your kitchen is well organized and you are
(C) If a performance by an actor in a play enhances a good cook, the meals you prepare will be
the audience’s appreciation of the play, then the delicious. The dinner Jon prepared last week
play as a whole is better as a result. was not at all tasty, so his kitchen must not be
(D) An effective dramatic performance in a play will well organized.
enhance the audience’s appreciation of the play. (C) If Bob starts exercising daily, he will improve his
(E) A dramatic performance that makes it more fitness level. His fitness level will also improve
difficult for the audience to empathize with the if he quits smoking. So if he refuses to exercise
actor’s character detracts from the audience’s and continues to smoke, his fitness level is not
appreciation of the play. going to improve.
(D) The police are allowed to ticket any parked car
23. The computer software industry is rapidly growing and that does not have a parking permit. They are
has been for some time. For a company to grow rapidly, also allowed to ticket any parked car whose
it must be innovative, which means it must have creative parking permit has expired. Thus, the police are
employees or a management that is open to new ideas. going to ticket Ruta’s car, for her parking permit
Logichut is a software company that is said to have no has expired.
creative employees. If so, its management is evidently (E) Most dogs that have attended obedience classes
open to new ideas. respond correctly to basic commands, but only
The reasoning in the argument is flawed in that it some are able to perform advanced tricks. Thus,
any dog that has attended obedience classes and
(A) illicitly presumes that because a set of things has performs advanced tricks will be able to respond
a certain property, each member of that set has correctly to basic commands.
the property
(B) confuses a necessary condition for a company’s
being innovative with a sufficient condition for a
company’s being innovative
(C) illicitly concludes that because many people
believe something, it must be true
GO ON TO THE NEXT PAGE.
(D) confuses a necessary condition for a company
to grow rapidly with a sufficient condition for a
company to grow rapidly
(E) overlooks the possibility that a software company
could have both creative employees and a
management that is open to new ideas
Law School Admission Council
4 4 4 4 4 -33-
4
25. Translator: Dr. Abner, apparently scornful of my 26. Letter to the editor: In calling on antinuclear activists to
translation of an ancient poem on the grounds “live up to their own ideals” and help find better
that it is inaccurate, says that my translation ways of disposing of nuclear waste, advocates of
produces in most readers feelings other than nuclear power merely prove their own dishonesty.
those the original text was intended to produce. Everyone knows that antinuclear activists
However, as Dr. Abner should realize, we do not want the world rid of nuclear power as soon
know how people originally responded to the as possible, and they believe any other option
poem or how its author intended them to respond. is environmentally irresponsible. But finding
So Dr. Abner’s criticism is unjustified. better ways of disposing of nuclear waste would
probably make nuclear power acceptable to more
The translator’s argument is flawed in that it
people and thus likelier to continue.
(A) fails to adequately address the possibility that the
Which one of the following principles, if valid, most
original poem had more than one author
helps to justify the reasoning in the argument?
(B) draws a conclusion purely on the basis of an
appeal to popular opinion (A) If other options are available, it is irresponsible to
(C) fails to adequately address the possibility that the take a possibly beneficial course of action that
original text was not intended to produce any has the potential of causing harm.
particular feelings in the reader (B) It is better to eliminate a potentially harmful
(D) rejects an argument merely because of the practice than to try to improve it, if improving it
circumstances of the person who offered it will not significantly reduce its capacity to cause
(E) concludes that we cannot know that certain harm.
effects were not what the author intended merely (C) If one is calling on people to act in a way that
because we do not know what specific effects will possibly undermine their goals, it is
the author did intend dishonest to ask that they do so for the sake of
their ideals.
(D) It is irresponsible to try to induce others to find a
practice acceptable if it is not one’s own belief
that that practice is acceptable.
(E) It is dishonest to ask people to live up to their
ideals without revealing whether one’s own
ideals conflict with theirs.
S T O P
IF YOU FINISH BEFORE TIME IS CALLED, YOU MAY CHECK YOUR WORK ON THIS SECTION ONLY.
DO NOT WORK ON ANY OTHER SECTION IN THE TEST.
Law School Admission Council
-34-
Acknowledgment is made to the following sources from which material has been adapted for use in this test:
Steven Ashley, “It’s Not Easy Being Green” in Scientifc American. ©2002 by Scientifc American, Inc.
Scott DeVeaux, The Birth of Bebop. ©1997 by The Regents of the University of California.
Freeman J. Dyson, “One in a Million” in New York Review of Books. ©2004 by NYREV, Inc.
Geoffrey W. G. Leane, “Testing Some Theories About Law” in Melbourne University Law Review. ©1995 by
Melbourne University Law Review Association.
“Shaking Up the Day.” ©1988 by Discover, Inc.
Michael Shermer, “Freeman Dyson, Miracles, and the Belief in the Paranormal” in eSkeptic newsletter. ©2004 by
The Skeptics Society and Michael Shermer.
John Timpane, “How to Convince a Reluctant Scientist” in Scientifc American. ©1995 by Scientifc American, Inc.
Law School Admission Council
-35-
PrepTest 94 Conversion Table
The table below should be used for converting raw scores to their corresponding 120–180 scaled scores for the July 2020
LSAT-Flex administration. The table below shows the LSAT-Flex reported score that corresponds to each possible raw score.
Raw Score Scaled Score Raw Score Scaled Score
76 180 37 146
75 180 36 146
74 179 35 145
73 177 34 144
72 175 33 143
71 174 32 142
70 173 31 142
69 171 30 141
68 170 29 140
67 169 28 139
66 168 27 138
65 167 26 137
64 167 25 136
63 166 24 135
62 165 23 134
61 164 22 133
60 163 21 132
59 163 20 131
58 162 19 130
57 161 18 129
56 160 17 128
55 160 16 126
54 159 15 125
53 158 14 123
52 158 13 121
51 157 12 120
50 156 11 120
49 155 10 120
48 155 9 120
47 154 8 120
46 153 7 120
45 152 6 120
44 152 5 120
43 151 4 120
42 150 3 120
41 149 2 120
40 149 1 120
39 148 0 120
38 147
Individual Performance on the Test: Scores are reported on a 120–180 score scale, with 120 being the lowest possible score and
180 being the highest possible score.
Law School Admission Council
-36-
PrepTest 94 Answer Key
Question Section 1 Section 2* Section 3 Section 4
1 D A B D
2 D B E C
3 A C B D
4 E E D C
5 A B A E
6 B D B E
7 E A A A
8 C E D B
9 B E D B
10 A C C D
11 D D E B
12 E C C D
13 C E E D
14 D D B B
15 A A A D
16 D A B D
17 B D C A
18 D C D C
19 E B D A
20 B C C B
21 D D D E
22 A C C E
23 B A C A
24 A D C
25 B C E
26 A C
27 D
*Section 2 is unscored. The number of items answered correctly in Section 2 should not be added to the raw score.
Law School Admission Council